Đến nội dung

tranductucr1

tranductucr1

Đăng ký: 28-04-2015
Offline Đăng nhập: 16-06-2019 - 22:46
*****

#659191 Đề thi học sinh giỏi 12 tỉnh Cà Mau 2016-2017

Gửi bởi tranductucr1 trong 24-10-2016 - 18:05

Nguồn: page Toán học Bắc Trung Nam

Đặt $t=\sin x+\cos x$ ta có $\sin 2x=2\sin x \cos x =t^2-1$
$f(x)$ trở thành : $f(x) =\frac{t}{t^2-t-1}$ 
Ta lại có $t=\sqrt{2}\sin(x+\frac{\pi}{4})$ => $1 \leq t \leq \sqrt{2}$
ta có $f'(x)=-\frac{t^2+1}{(t^2-t-1)^2} <0$ với mọi $t$
Vậy hàm số luôn nghịch biến từ đó ta có $f(\sqrt{2}) \leq f(x) \leq f(1)$




#659172 $2x+\frac{y}{\sqrt{4x^{2}+1...

Gửi bởi tranductucr1 trong 24-10-2016 - 13:15

$2x+\frac{y}{\sqrt{4x^{2}+1}+2x}+y^{2}=0$ và $4(\frac{x}{y})^2+2\sqrt{4x^2+1}+y^2=3$

Có thể theo giải theo hướng này 
$4(\frac{x}{y}^2+1+2y\sqrt{4\frac{x^2}{y^2}+1}+y^2=3$
$(\sqrt{4\frac{x^2}{y^2}+1}+y)^2=3$ 
=>...




#659153 ĐỀ THI HSG LỚP 12 TỈNH BÌNH ĐỊNH NĂM 2016-2017

Gửi bởi tranductucr1 trong 24-10-2016 - 05:21

bài toán Bernouli - Euler  là gì vậy :)) em tìm trên Google mà không thấy anh ạ

http://www.bachkhoat...bo-nham-thu.htm có thể là cái này chăng




#658767 P=$\frac{4a}{b+c-a}+\frac{9b}...

Gửi bởi tranductucr1 trong 22-10-2016 - 11:54

Cho a, b,c là độ dài ba cạnh của một tam giác. Tìm giá trị nhỏ nhất của biểu thức:

      P=$\frac{4a}{b+c-a}+\frac{9b}{a+c-b}+\frac{16c}{a+b-c}$

$P=\frac{4a}{b+c-a}+\frac{9b}{a+c-b}+\frac{16c}{a+b-c}$
$P=\frac{4a}{b+c-a}+2)+\frac{9b}{a+c-b}+\frac{9}{2}+\frac{16c}{b+a-c}+8-(8+2+\frac{9}{2})$
$P=2\frac{a+b+c}{b+c-a}+\frac{9(a+b+c)}{2(a+c-b)}+\frac{8(a+b+c)}{b+a-c}-(8+2+\frac{9}{2})$
$P=\frac{a+b+c}{2}(\frac{4}{b+c-a}+\frac{9}{a+c-b}+\frac{16}{b+a-c}) -(8+2+\frac{9}{2}) \geq \frac{a+b+c}{2} \frac{(2+3+4)^2}{a+c+b} -(8+2+\frac{9}{2}) \geq ...$




#658696 $a^{2}(b+c-a)+b^{2}(c+a-b)+c^{2}(a+b-c)...

Gửi bởi tranductucr1 trong 21-10-2016 - 19:43

Cho a,b,c là 3 cạnh tam giác.Chứng minh rằng

$a^{2}(b+c-a)+b^{2}(c+a-b)+c^{2}(a+b-c)\leq 3abc$

Ta có biến đổi 
$a^2(c+b-a)+b^2(a+c-b)+c^2(a+b-c) \leq 3abc$
<=> $ a(b^2+c^2)+c(a^2+b^2)+b(a^2+c^2)-3abc-a^3-b^3-c^3\leq 0$
<=> $a(b-c)^2+c(a-b)^2+b(a-c)^2 +(3abc-a^3-b^3-c^3) \leq 0$
<=> $P= (a+b-c)(a-b)^2+(b+c-a)(b-c)^2+(a+c-b)(a-c)^2 \geq 0$
Đặt $S_c=a+b-c,S_b=a+c-b,S_a=b+c-a$
Không mất tính tổng quát ta có thể giả sự $a\geq b \geq c$
=> $S_b ,S_c \geq 0$
ta có $(a-c)^2 \geq (a-b)^2+(b-c)^2$ <=> $ac+b^2-ab-ac \leq 0$ <=> $(b-c)(b-a) \leq 0$(đúng )
Vậy $P\geq (S_b+S_c)(a-b)^2+(S_b+S_a)(b-c)^2 \geq 0 $(Dpcm ) vì $S_a+S_b=a+c-b+b+c-a=2c \geq 0$ 
Bài toán được chứng minh hoàn tất




#658687 BT BĐT trong tài liệu chuyên toán đại số 10

Gửi bởi tranductucr1 trong 21-10-2016 - 18:11

attachicon.gifCapture.PNG

attachicon.gifbdt.PNG

 

Bài 8 và 23 vẫn chưa có lời giải!!! 

#1
Chuẩn hóa $abc=1$ 
ta có $\frac{1}{a^3+b^3+1} \leq \frac{a+b+c^4}{(a^2+b^2+c^2)} $
cần chứng minh  $(a^2+b^2+c^2)^2 \geq 2(a+b+c)+a^4+b^4+c^4$
<=> $ \sum (ab)^2 \geq \sum a$
ta lại có $(ab)^2+(bc)^2+(ac)^2 \geq abc(a+b+c)=a+b+c$ dpcm


 




#658685 BT BĐT trong tài liệu chuyên toán đại số 10

Gửi bởi tranductucr1 trong 21-10-2016 - 18:01

attachicon.gifCapture.PNG

attachicon.gifbdt.PNG

 

Bài 8 và 23 vẫn chưa có lời giải!!! 

Bài 13 áp dùng bất đẳng thức $z^2+y^2+yz \geq \frac{3(z+y)^2}{4}$
Ta có $VP \geq \frac{xy+yz+xz}{\sqrt{3}(x+y+z)}$
vậy ta chỉ cần chứng minh 
$(x+y+z)^2 \geq 3(xz+yz+xz)$( Đây là bất đẳng thức cơ bản )




#658662 Cho x,y,z>0 thỏa mãn: x+y+z=1. Tìm max: P=$\sqrt{\fra...

Gửi bởi tranductucr1 trong 21-10-2016 - 11:55

Cho x, y, z>0 thỏa mãn: x+y+z=1. Tìm max:

P=$\sqrt{\frac{xy}{z+xy}}+\sqrt{\frac{yz}{x+yz}}+\sqrt{\frac{zx}{y+zx}}$

$P=\Sigma \sqrt{\frac{xy}{z(x+y+z)+xy}}=\Sigma \sqrt{\frac{xy}{(x+z)(y+z)}} \leq \frac{1}{2}*\Sigma (\frac{x}{x+z}+\frac{y}{y+z})=\frac{3}{2}$
Dấu bằng xảy ra khi và chỉ khi $x=y=z=\frac{1}{3}$




#649742 Tìm giá trị lớn nhất của A

Gửi bởi tranductucr1 trong 15-08-2016 - 13:37

Tìm GTLN của A=$\sqrt{x+1}-\sqrt{x-8}$

Tìm GTNN của B=$\sqrt{x-3 }$

$\sqrt{x+1}-\sqrt{x-8}=\frac{9}{\sqrt{x+1}+\sqrt{x-8}} \leq 1$ (vì $x \geq 8$ ) 
b) GTNN của B=0 :v 




#645153 $\frac{1}{2}+\frac{1}{3...

Gửi bởi tranductucr1 trong 16-07-2016 - 13:05

chứng minh $\frac{1}{2}+\frac{1}{3\sqrt{2}}+....+\frac{1}{(x+1)\sqrt{x}}<2$

Dễ dàng chứng minh bdt đúng với $x=1$
 Với $x \geq 2$ Ta sẽ chứng minh bdt mạnh hơn là 
$\frac{1}{2}+..+\frac{1}{(x+1)\sqrt{x}} < 2-\frac{2}{\sqrt{x^3}}  (1) $
Với $x=1 ,x=2,x=3 $ (1) đều đúng 

Giả sứ $(1)$ đúng với $x=k$ ($k$ thuộc tập số tự nhiên )
$\frac{1}{2}+...+\frac{1}{(k+1)\sqrt{k}} < 2-\frac{2}{\sqrt{k^3}} (2) $
Ta sẽ chứng minh bdt đúng với $x=k+1$
Thật vậy
$\frac{1}{2}+...+\frac{1}{(k+2)\sqrt{k+1}} < 2-\frac{2}{\sqrt{k^3}}+\frac{1}{(k+2)\sqrt{k+1}}<2-\frac{2}{\sqrt{(k+1)^3}}$
Vậy t chỉ cần chỉ ra được 
$\frac{2}{\sqrt{k^3}}-\frac{2}{\sqrt{(k+1)^3}} >\frac{1}{(k+2)\sqrt{k+1}}$
<=> $2\frac{\sqrt{(k+1)^3}-\sqrt{k^3}}{\sqrt{k^3(k+1)^3}} >\frac{1}{(k+2)\sqrt{k+1}}$
<=> $2(\sqrt{(1+\frac{1}{k})^3}-1)>\frac{k+1}{k+2}$(3) 
Đặt $a=\frac{1}{k}$ ($\frac{1}{2}<a<1$)
(3) <=> $2>\frac{a+1}{1+2a}*\frac{1}{\sqrt{(1+a)^3}-1}$
Đúng vì $\frac{1}{2}<a<1$
bài toán được chứng minh hoàn toàn 




#644659 CMR: $\dfrac{1 + \sqrt[]{1 + x^2}}{x...

Gửi bởi tranductucr1 trong 12-07-2016 - 15:00

Cho x,y,z dương thỏa mãn: $\frac{1}{xy}+\frac{1}{yz}+\frac{1}{zx}=1$. Chứng minh rằng:

$\dfrac{1 + \sqrt[]{1 + x^2}}{x} + \dfrac{1 + \sqrt[]{1 + y^2}}{y} + \dfrac{1 + \sqrt[]{1 + z^2}}{z} \le xyz$

Ta có $xyz(x+y+z)=(x+y+z)^2 \geq 3(xy+yz+xz)$ (Vì $x+y+z=xyz$)
=> $xyz \geq  3 (\frac{1}{x}+\frac{1}{y}+\frac{1}{z})$

=> $xyz \geq \sum \frac{1}{x} +\sum (\frac{1}{x}+\frac{1}{y})$
=> $ xyz  \geq \sum \frac{1}{x} +\sum \sqrt{(\frac{1}{x}+\frac{1}{y})(\frac{1}{x}+\frac{1}{z})}$
=> $xyz \geq \sum \frac{1}{x} +\sum \sqrt{\frac{1}{xy}+\frac{1}{yz}+\frac{1}{xz}+\frac{1}{x^2}}$

=>$xyz \geq \sum \frac{1}{x} +\sum \sqrt{\frac{1}{x^2} +1}$
=> $Dpcm$
 




#639492 Cho $\alpha\ge \sqrt{2}$ và $q>0...

Gửi bởi tranductucr1 trong 10-06-2016 - 23:56

Cho $\alpha\ge \sqrt{2}$ và $q>0$. Chứng minh rằng: $(q^{\alpha}+q^{-\alpha})^2\ge 2(q^2+q^{-2})$

Ta có    $(q^{\alpha}+q^{-\alpha})^2 \geq 2(q^{\alpha}+q^{-\alpha})$

 Bất đẳng thức sẽ đúng nếu ta chứng minh được 

$q^{2\alpha}+q^{-2\alpha} \geq q^{2}+q^{-2}$

$\Leftrightarrow q^{2}(q^{\alpha}-1)+q^{-2}\frac{(q^{\alpha}-1)}{q^{\alpha}} \geq 0$

$\Leftrightarrow (q^{\alpha}-1)\frac{(q^{4\alpha}-1)}{q^{4\alpha}} \geq 0$

$\Leftrightarrow (q^{\alpha}-1)^{2}\frac{(q^{3\alpha}+..+1)}{q^{4\alpha}} \geq 0$ 

Điều kiện hơi thừa nhỉ :v :v :v 

 




#639484 $\sum a \geq 3\sum \frac{1}{a}...

Gửi bởi tranductucr1 trong 10-06-2016 - 23:25

$gt \Leftrightarrow \sum \frac{a^2-3}{1+a^2}=0$

bđt $\Leftrightarrow \sum \frac{a^2-3}{a} \ge 0 \leftrightarrow \sum \frac{(a^2-3)(1+a^2)}{(1+a^2)a} \ge 0$

giả sử $a \ge b \ge c \ge d$ suy ra $f(x)=\frac{x^2-3}{x^2+1}$ và $g(x)=\frac{1+x^2}{x}$ là các hàm đơn điệu tăng trên $R^+$

áp dụng $Chebyshev$ cho $2$ bộ trên ta được đpcm

Chưa chắc gì $\frac{1+x^2}{x}$ đã là hàm đơn điệu bạn cần chứng minh !!! :) 




#633641 $\left\{\begin{matrix} x-\frac{1...

Gửi bởi tranductucr1 trong 17-05-2016 - 14:03

Giải HPT: $\left\{\begin{matrix} x-\frac{1}{x}=y-\frac{1}{y} & \\ 3{{x}^{2}}+{{y}^{2}}-2y-2=0 & \end{matrix}\right.$

Từ (1)  suy ra $(x-y)(1-\frac{1}{xy})=0$

TH1 x=y  thay vào giải (2) 

TH2 xy=1 ta có $x=\frac{1}{y}$ thay vào (2) giải phương trình bậc 4 


  • NAT yêu thích


#633291 Bài tập Chứng minh bất đẳng thức

Gửi bởi tranductucr1 trong 15-05-2016 - 17:49

1) Cho $z\geq y\geq x> 0$. Chứng minh: $y(\frac{1}{x}+\frac{1}{y})+\frac{1}{y}(x+z)\leq (x+z)(\frac{1}{x}+\frac{1}{z})$

2) Cho a, b, c dương và abc=1. Chứng minh: $\frac{1}{a^{2}(b+c)}+\frac{1}{b^{2}(a+c)}+\frac{1}{c^{2}(b+a)}\geq \frac{3}{2}$

                                                                   và $\frac{x}{y+z}+\frac{y}{x+z}+\frac{z}{x+y}\geq \frac{3}{2}$

3) Với a, b, c là 3 cạnh một tam giác. Chứng minh rằng $a(b-c)^{2}+b(c-a)^{2}+c(a+b)^{2}> a^{3}+b^{3}+c^{3}$

4) Cho a, b, c, d dương.

Chứng minh: $\sqrt{(a+b)(c+d)}+\sqrt{(a+c)(b+d)}+\sqrt{(a+d)(b+c)}\geq \sqrt{ab}+\sqrt{bc}+\sqrt{cd}+\sqrt{da}+\sqrt{ac}+\sqrt{bd}$.

5) Chứng minh: $\frac{a^{2}+b^{2}+c^{2}}{3}\geq (\frac{a+b+c}{3})^{2}$

6) Cho a, b, c dương. Chứng minh: $\frac{a}{a+b}+\frac{b}{b+c}+\frac{c}{a+c}> 1$

7) Với 0<x<y<z, chứng minh $\frac{x^{2}}{z}<\frac{x^{2}+y^{2}+z^{2}}{x+y+z}<\frac{z^{2}}{x}$

8) Với a, b, c là các số dương. Chứng minh rằng $a^{2}(b+c-a)+b^{2}(c+a-b)+c^{2}(a+b-c)\leq 3abc$

9) Với a, b, c dương. Chứng minh:

a) $(a+b-c)(a+c-b)(b+c-a)\leq abc$

b) $\frac{a^{3}}{a^{2}+ab+b^{2}}+\frac{b^{3}}{b^{2}+bc+c^{2}}+\frac{c^{3}}{c^{2}+ac+a^{2}}\geq \frac{a+b+c}{3}$

10) Với a, b, c là độ dài 3 cạnh một tam giác. Chứng minh rằng:

a) $\frac{a}{b+c-a}+\frac{b}{a+c-b}+\frac{c}{a+b-c}\geq 3$

b) $\frac{a^{2}}{b+c-a}+\frac{b^{2}}{a+c-b}+\frac{c^{2}}{a+b-c}\geq a+b+c$

(Mình cần lời giải của 10 bài tập trên cộng thêm với 10 bài trong 3 ảnh gửi kèm, rất gấp, ai giúp mình với, ai biết thì xin nán lại giúp mình - BÀI NÀO LÀM RỒI MÌNH SẼ TÔ ĐỎ HOẶC ĐÁNH DẤU CHỮ "R")

 

1) Cho $z\geq y\geq x> 0$. Chứng minh: $y(\frac{1}{x}+\frac{1}{y})+\frac{1}{y}(x+z)\leq (x+z)(\frac{1}{x}+\frac{1}{z})$

2) Cho a, b, c dương và abc=1. Chứng minh: $\frac{1}{a^{2}(b+c)}+\frac{1}{b^{2}(a+c)}+\frac{1}{c^{2}(b+a)}\geq \frac{3}{2}$

                                                                   và $\frac{x}{y+z}+\frac{y}{x+z}+\frac{z}{x+y}\geq \frac{3}{2}$

3) Với a, b, c là 3 cạnh một tam giác. Chứng minh rằng $a(b-c)^{2}+b(c-a)^{2}+c(a+b)^{2}> a^{3}+b^{3}+c^{3}$

4) Cho a, b, c, d dương.

Chứng minh: $\sqrt{(a+b)(c+d)}+\sqrt{(a+c)(b+d)}+\sqrt{(a+d)(b+c)}\geq \sqrt{ab}+\sqrt{bc}+\sqrt{cd}+\sqrt{da}+\sqrt{ac}+\sqrt{bd}$.

5) Chứng minh: $\frac{a^{2}+b^{2}+c^{2}}{3}\geq (\frac{a+b+c}{3})^{2}$

6) Cho a, b, c dương. Chứng minh: $\frac{a}{a+b}+\frac{b}{b+c}+\frac{c}{a+c}> 1$

7) Với 0<x<y<z, chứng minh $\frac{x^{2}}{z}<\frac{x^{2}+y^{2}+z^{2}}{x+y+z}<\frac{z^{2}}{x}$

8) Với a, b, c là các số dương. Chứng minh rằng $a^{2}(b+c-a)+b^{2}(c+a-b)+c^{2}(a+b-c)\leq 3abc$

9) Với a, b, c dương. Chứng minh:

a) $(a+b-c)(a+c-b)(b+c-a)\leq abc$

b) $\frac{a^{3}}{a^{2}+ab+b^{2}}+\frac{b^{3}}{b^{2}+bc+c^{2}}+\frac{c^{3}}{c^{2}+ac+a^{2}}\geq \frac{a+b+c}{3}$

10) Với a, b, c là độ dài 3 cạnh một tam giác. Chứng minh rằng:

a) $\frac{a}{b+c-a}+\frac{b}{a+c-b}+\frac{c}{a+b-c}\geq 3$

b) $\frac{a^{2}}{b+c-a}+\frac{b^{2}}{a+c-b}+\frac{c^{2}}{a+b-c}\geq a+b+c$

(Mình cần lời giải của 10 bài tập trên cộng thêm với 10 bài trong 3 ảnh gửi kèm, rất gấp, ai giúp mình với, ai biết thì xin nán lại giúp mình - BÀI NÀO LÀM RỒI MÌNH SẼ TÔ ĐỎ HOẶC ĐÁNH DẤU CHỮ "R")

3. áp dụng bất đẳng thức Amgm cho 2004 t có  $a^{2004}+a^{2004}+...+a^{2004}+1 \geq 2004 \sqrt[2004]{a^{2004*2003}}=2004a^{2003}$ 
Từ đó dễ dàng có được $2003(a^{2004}+b^{2004}+c^{2004}) +3 \geq 2004(a^{2003}+b^{2003}+c^{2003}) \geq 2003(a^{2003}+b^{2003}+c^{2003})+3 $
vì  từ amgm dễ dàng chứng minh được $a^{2003}+b^{2003}+c^{2003} \geq 3$

từ đó ta có $2003(a^{2004}+b^{2004}+c^{2004}) \geq 2003(a^{2003}+b^{2003}+c^{2003})$ 
=>...